Solving Fibonacci Function Homework: Prove f(x)=\frac{1+x}{(1-x-x^{2})}

  • Thread starter Thread starter talolard
  • Start date Start date
  • Tags Tags
    Function
Click For Summary
The discussion centers on proving the generating function for the Fibonacci sequence, expressed as f(x) = (1 + x) / (1 - x - x^2). The initial setup involves defining f(x) as the sum of Fibonacci numbers multiplied by powers of x. A key mistake identified is in the summation of terms, where the correct expression for the series starting from n=2 should be f(x) - 1, rather than f(x). This correction leads to the proper formulation of the equation, ultimately confirming the generating function. The conversation highlights the importance of accurately handling series in mathematical proofs.
talolard
Messages
119
Reaction score
0

Homework Statement



I am making a mistake and i can't find it. Pleasde help me


Let \left\{ a_{n}\right\} be the fibonacci sequence and f(x)=\sum a_{n}x^{n}. Prove that [/tex] f(x)=\frac{1}{1-x-s^{2}} [/tex]in its radius of convergence.

Solution

f(x)=\sum a_{n}x^{n}=1+x+\sum_{n=2}a_{n}x^{n}=1+x+\sum_{n=2}a_{n-1}x^{n}+\sum_{n=2}a_{n-2}x^{n}=1+x+x\sum_{n=2}a_{n-1}x^{n-1}+x^{2}\sum_{n=2}a_{n-2}x^{n-2}=1+x+xf(x)+x^{2}f(x)

=f(x)(x+x^{2})+1+x=f(x)\iff f(x)(1-x-x^{2})=1+x\iff f(x)=\frac{1+x}{(1-x-x^{2})}

Homework Equations





The Attempt at a Solution


 
Physics news on Phys.org
I don't know if it will solve all your problems, but you claim that
\sum_{n = 2}^\infty a_{n - 1} x^{n - 1} = f(x)
However, I think it is
\sum_{n = 2}^\infty a_{n - 1} x^{n - 1} = f(x) - 1
 
Actually., I think that exactly solves my problem.
1+x+x\sum_{n=2}a_{n-1}x^{n-1}+x^{2}\sum_{n=2}a_{n-2}x^{n-2}=1+x+xf(x)-x+x^{2}f(x)=1+f(x)(x+x^{2})=f(x)\iff f(x)(1-x-x^{2})=1\iff=\frac{1}{1-x-x^{2}}
THanks
 
Question: A clock's minute hand has length 4 and its hour hand has length 3. What is the distance between the tips at the moment when it is increasing most rapidly?(Putnam Exam Question) Answer: Making assumption that both the hands moves at constant angular velocities, the answer is ## \sqrt{7} .## But don't you think this assumption is somewhat doubtful and wrong?

Similar threads

  • · Replies 14 ·
Replies
14
Views
2K
  • · Replies 2 ·
Replies
2
Views
2K
  • · Replies 13 ·
Replies
13
Views
2K
  • · Replies 5 ·
Replies
5
Views
2K
  • · Replies 7 ·
Replies
7
Views
2K
  • · Replies 11 ·
Replies
11
Views
3K
  • · Replies 3 ·
Replies
3
Views
950
  • · Replies 3 ·
Replies
3
Views
2K
  • · Replies 4 ·
Replies
4
Views
1K
Replies
8
Views
2K